Is the magnetic field from this current rotationally symmetric?

  • #1
Trollfaz
137
14
Imagine a very large disc with current converging to it's center as $$ I=- \frac{k}{r} \hat{r}$$
Obviously ##\theta## is absent in the equation so the magnetic field by this current should look the same no matter which angle one is observing. Then does this mean that the magnetic field is dependent on r and exists as circles (anticlockwise above the disc, clockwise below the disc) and weakens with r under Biot Savart Law
 
Physics news on Phys.org
  • #2
Your current density violates conservation of charge, which follows directly from Maxwell’s equations.
 
  • Like
Likes vanhees71 and Vanadium 50
  • #3
Conservation of charge is only true if the system is isolated entirely but this can be maintained by external sources constantly inputting or withdrawing charges
 
  • #4
Trollfaz said:
Conservation of charge is only true if the system is isolated entirely but this can be maintained by external sources constantly inputting or withdrawing charges
When I say conservation of charge I refer to the continuity equation for charge. This is a local statement and your setup is violating it.
 
  • Like
Likes vanhees71
  • #5
Trollfaz said:
Conservation of charge is only true if the system is isolated entirely but this can be maintained by external sources constantly inputting or withdrawing charges
That inputting or withdrawing charge is a current.

The conservation of charge follows from Maxwell’s equations. If it is violated then Maxwell’s equations do not hold. Without them is there even a magnetic field at all? Certainly Biot Savart would not apply either.
 
  • #6
This reminds me of "Can God create a stone so heavy he can't lift it?"

Asking what Maxwell's equations say for a system that violates them (and of course, cannot be physically realized) is pointless. Some would use other adjectives.
 
  • Like
Likes Dale and vanhees71
  • #7
First of all a current is a scalar. What you seem to mean is a current density. Now you write
$$\vec{j}=-\frac{k}{r} \hat{r}.$$
This is obviously a stationary current density, and thus you should have ##\vec{\nabla} \cdot \vec{j}=0##, but for your field
$$\vec{\nabla} \cdot \vec{j} = -\frac{k}{r^2} \neq 0,$$
i.e., there's no solution for the magnetostatics problem,
$$\vec{\nabla} \times \vec{B} = \mu_0 \vec{j}.$$
Maxwell's equations in general are only solvable, when the electromagnetic charge is conserved, i.e., if the continuity equation,
$$\partial_t \rho + \vec{\nabla} \cdot \vec{j}=0,$$
holds. This "integrability condition" follows from the gauge invariance of Maxwell's theory.
 
  • #8
vanhees71 said:
This is obviously a stationary current density, and thus you should have ∇→⋅j→=0, but for your field
∇→⋅j→=−kr2≠0,
i.e
This would be true in spherical coordinates. However, the OP is referring to a disc, which implies ##r## should probably be interpreted as the cylinder coordinate radius. If so, ##\nabla\cdot\vec j=0## everywhere … except in ##r=0##, where it is not.
 
  • Like
Likes Vanadium 50 and vanhees71
  • #9
Well, ill-defined problems can lead easily to misunderstandings. So what's meant then seems to be
$$\vec{j}=-\frac{k}{R} \hat{R} \delta(z),$$
where ##(R,\varphi,z)## are cylinder coordinates. That also obviously violates ##\vec{\nabla} \cdot \vec{j}=0## ;-)).
 
  • #10
vanhees71 said:
Well, ill-defined problems can lead easily to misunderstandings. So what's meant then seems to be
$$\vec{j}=-\frac{k}{R} \hat{R} \delta(z),$$
where ##(R,\varphi,z)## are cylinder coordinates. That also obviously violates ##\vec{\nabla} \cdot \vec{j}=0## ;-)).
Not as obvious as you might think. The only violation occurs in R=0 where the coordinates are singular. Since the delta function depends only on z, it will not contribute to any divergence. The divergence becomes ##[\partial_R(-k\delta(z))]/R = 0## for ##R>0##. It takes an additional argument to conclude the non-zero divergence at the origin so I’m not sure I would call it obvious.
 
  • #11
So the Biot Savart Law and Ampere's law only applies to steady current. Is the definition of a steady current one that results in no charge to the system
 
  • #12
Time independent and ##\nabla\cdot\vec j =0##, which leads to no charge accumulation. Somewhat simplified: charge accumulation would mean a changing charge, which would mean a changing E field, which would give additional contributions to the B field.
 
  • Like
Likes Dale
  • #13
Orodruin said:
Not as obvious as you might think. The only violation occurs in R=0 where the coordinates are singular. Since the delta function depends only on z, it will not contribute to any divergence. The divergence becomes ##[\partial_R(-k\delta(z))]/R = 0## for ##R>0##. It takes an additional argument to conclude the non-zero divergence at the origin so I’m not sure I would call it obvious.
To see what happens at ##R=0## let's integrate over an arbitrary collinear cylinder ##Z## of radius ##a##. You formally get
$$\int_Z \mathrm{d}^3 x \vec{\nabla} \cdot \vec{j} = \int_{\partial Z} \mathrm{d}^2 \vec{f} \cdot \vec{j}.$$
The bottom and top circles don't contribute, and the mantle area element normal vector is ##\mathrm{d}^2 \vec{f}=a \hat{R} \mathrm{d} \varphi \mathrm{d} z##:
$$\int_{M} \mathrm{d}^2 \vec{f} \cdot \vec{j}=\int_{0}^{2 \pi} \mathrm{d} \varphi \int_{-h/2}^{h/2} \mathrm{d} z a (-k/a) \delta(z)=-2 \pi k \neq 0.$$
So, as is also intuitively obvious, the singularity is such that there is a non-zero current, leading to an accumulation of charge at the center of the disk. I'm a bit lost at the task to find the electromagnetic fields. You have to assume some singular time-dependent charge density to get the charge conservation right...
 

1. Is the magnetic field from this current rotationally symmetric?

Yes, the magnetic field from a current is rotationally symmetric if the current distribution is also rotationally symmetric. This means that the magnetic field strength and direction will be the same at all points equidistant from the current-carrying wire.

2. How can I determine if the magnetic field from a current is rotationally symmetric?

You can determine if the magnetic field from a current is rotationally symmetric by analyzing the current distribution. If the current is evenly distributed around a central axis or point, then the resulting magnetic field will be rotationally symmetric.

3. What are the implications of having a rotationally symmetric magnetic field?

Having a rotationally symmetric magnetic field means that the magnetic field strength and direction will be uniform in all directions around the current-carrying wire. This can simplify calculations and make it easier to predict the behavior of magnetic materials in the vicinity of the current.

4. Are there any real-world applications where a rotationally symmetric magnetic field is important?

Yes, there are many real-world applications where a rotationally symmetric magnetic field is important. For example, in MRI machines, having a rotationally symmetric magnetic field is crucial for producing clear and accurate images of the human body.

5. Can a non-rotationally symmetric current produce a rotationally symmetric magnetic field?

No, a non-rotationally symmetric current distribution will not produce a rotationally symmetric magnetic field. The resulting magnetic field will reflect the asymmetry of the current distribution, leading to varying field strengths and directions at different points around the current-carrying wire.

Similar threads

Replies
8
Views
681
Replies
4
Views
286
Replies
4
Views
988
Replies
1
Views
1K
Replies
1
Views
963
Replies
15
Views
2K
Replies
5
Views
877
Replies
2
Views
784
  • Introductory Physics Homework Help
Replies
12
Views
331
Replies
5
Views
1K
Back
Top